Genetic Disorders [Robbins and Cotran Review of Pathology (4E) CH5/Rubins Illustrated Pathology Review (2E) CH6]

Lakukan tugas rumah & ujian kamu dengan baik sekarang menggunakan Quizwiz!

Which of the following best describes the pathogenesis of hepatosplenomegaly and bone pain in a patient with Gaucher disease? (A) Accumulation of unmetabolized substrate (B) Decreased utilization of metabolic end-product (C) Formation of an abnormal metabolic end-product (D) Increased utilization of metabolic end-product (E) Opening of mitochondrial membrane pore

(A) Accumulation of unmetabolized substrate Glucosylceramide is a common core structure for membrane glycosphingolipids. The glucosylceramide that accumulates in Gaucher cells in the spleen, liver, bone marrow, and lymph nodes derives principally from the catabolism of senescent leukocytes. The membranes of these cells are rich in the cerebrosides, and when their degradation is blocked by the deficiency of glucocerebrosidase, the intermediate metabolite, glucosylceramide, accumulates. The other biochemical pathways listed do not cause sphingolipid accumulation in patients with lysosomal storage diseases. Dx: Gaucher disease

Which of the following best describes the pathogenesis of mental retardation in a patient with Phenylketonuria? (A) Accumulation of unmetabolized substrate (B) Decreased utilization of metabolic end-product (C) Formation of an abnormal metabolic end-product (D) Increased utilization of metabolic end-product (E) Opening of mitochondrial membrane pore

(A) Accumulation of unmetabolized substrate Phenylalanine is an essential amino acid that is derived exclusively from the diet and is oxidized in the liver to tyrosine by phenylalanine hydroxylase (PAH). A deficiency in PAH results in both hyperphenylalaninemia and the formation of phenylketones from the transamination of phenylalanine. The excretion in the urine of phenylpyruvic acid and its derivatives accounts for the original name of phenylketonuria. However, it is now established that phenylalanine itself, rather than its metabolites, is responsible for the neurologic damage central to this disease. Thus, the term hyperphenylalaninemia is actually a more appropriate designation than PKU. None of the other choices explains the accumulation of phenylalanine in these patients. Dx: Phenylketonuria

A 12-month-old boy shows progressive weakness, mental deterioration, and loss of vision. Laboratory studies demonstrate decreased activity of hexosaminidase A. The child eventually becomes blind and dies at 3 years of age. Which of the following best describes the pathogenesis of neuronal degeneration in this patient? (A) Accumulation of unmetabolized substrate (B) Decreased utilization of metabolic end-product (C) Formation of an abnormal metabolic end-product (D) Opening of mitochondrial membrane pore (E) Synthesis of a novel glycosphingolipid

(A) Accumulation of unmetabolized substrate Tay-Sachs disease is the catastrophic infantile variant of a class of lysosomal storage diseases known as GM2 gangliosidoses. This ganglioside is deposited in neurons of the central nervous system due to a failure of lysosomal degradation and accumulation of an unmetabolized substrate. Gangliosides are glycosphingolipids that are present in the outer leaflet of the plasma membrane, particularly in neurons. The lysosomal catabolism of ganglioside GM2 is accomplished through the activity of the β-hexosaminidases (A and B), which are composed of α and β subunits and require the participation of the GM2-activator protein. A deficiency in any of these components results in clinical disease. None of the other choices explains the pathogenesis of this disease. Dx: Tay-Sachs disease

A 4-year-old girl presents for a preschool physical examination. The child has a small head circumference, thin upper lip, and low-bridge nose. She shows evidence of mild mental retardation. Her parents state that she is often "emotional." Which of the following maternal causes of birth defects most likely accounts for these clinicopathologic findings? (A) Alcohol abuse (B) Cigarette smoking (C) Congenital syphilis (D) Inadequate nutrition (E) Poorly controlled diabetes mellitus

(A) Alcohol abuse Fetal alcohol syndrome refers to a complex of abnormalities induced by the maternal consumption of alcoholic beverages while pregnant that includes (1) growth retardation, (2) dysfunction of the central nervous system, and (3) characteristic facial dysmorphology (e.g., small head circumference and thin upper lip). One fifth of children with fetal alcohol syndrome have IQs below 70, and 40% have IQs between 70 and 85. Even with a normal IQ, these children tend to have short memory spans, impulsiveness, and emotional instability. The children of mothers who smoke (choice B) or who have inadequate nutrition (choice D) may also exhibit deficiencies in physical growth and intellectual development; however, there is no association with the pattern of facial dysmorphology and emotional instability seen in this case. Congenital syphilis (choice C) may also cause mental retardation, but it would show protean manifestations not illustrated in this case. Gestational diabetes (choice E) does not cause mental retardation. Dx: Fetal alcohol syndrome

As an adult, the brain of a patient with Down Syndrome will show histopathologic changes that are seen in patients with which of the following neurologic diseases? (A) Alzheimer disease (B) Huntington disease (C) Krabbe disease (D) Multiple sclerosis (E) Parkinson disease

(A) Alzheimer disease One of the most intriguing neurologic features of Down syndrome is its association with Alzheimer disease. The morphologic lesions characteristic of Alzheimer disease progress in all patients with Down syndrome and are universally demonstrable by age 35 years. These changes in the brain include (1) granulovacuolar degeneration, (2) neurofibrillary tangles, (3) senile plaques, and (4) loss of neurons. The senile plaques and cerebral blood vessels of both Alzheimer disease and Down syndrome always contain an amyloid composed of the same fibrillar protein (β-amyloid protein). The other choices are unrelated to Down syndrome. Dx: Down syndrome

35 A 3-year-old boy dies in an automobile accident. At autopsy, the right lung is markedly shrunken. Dissection shows that the right main stem bronchus ends blindly in nondescript tissue composed of rudimentary ducts and connective tissue. This finding represents an example of which of the following errors of morphogenesis? (A) Aplasia (B) Atresia (C) Dysraphicanomaly (D) Hypoplasia (E) Involutionfailure

(A) Aplasia Aplasia is the absence of an organ coupled with persistence of the organ anlage or a rudiment. Thus, aplasia of the lung refers to a condition in which the main bronchus ends blindly in nondescript tissue composed of rudimentary ducts and connective tissue. Dysraphic anomalies (choice C) are defects caused by the failure of apposed structures to fuse. Hypoplasia (choice D) refers to reduced size owing to the incomplete development of all or part of an organ. Examples include microphthalmia and microcephaly. Involution failures (choice E) reflect the persistence of embryonic or fetal structures that should involute at certain stages of development. A persistent thyroglossal duct is the result of incomplete involution of the tract that connects the base of the tongue with the thyroid. Dx: Pulmonary aplasia

Marfan Syndrome follows which of the following patterns of inheritance? (A) Autosomal dominant (B) Autosomal recessive (C) Multifactorial (D) X-linked dominant (E) X-linked recessive

(A) Autosomaldominant Marfan syndrome is an autosomal dominant, inherited disorder of connective tissue characterized by a variety of abnormalities in many organs, including the heart, aorta, skeleton, eyes, and skin. One third of cases represent sporadic mutations. The incidence in the United States is 1 per 10,000. Dx: Marfan syndrome

What will be the likely cause of death in a patient with Duchenne muscular dystrophy? (A) Cardiomyopathy (B) Cerebrovascular disease (C) End-stage renal disease (D) Pulmonary saddle embolism (E) Respiratory insufficiency

(A) Cardiomyopathy The symptoms of Duchenne muscular dystrophy (DMD) progress with age. During the first year of life, the infants appear normal, but more than half fail to walk by 18 months of age. More than 90% of afflicted boys are wheelchair bound by the age of 11 years. In advanced disease, cardiac symptoms are almost universal, and cardiomyopathy is a common cause of death. The mean age at death in boys with DMD is 17 years. The other choices are unrelated to DMD. Dx: Duchenne muscular dystrophy

An 87-year-old woman dies peacefully in her sleep. At autopsy, a rest of pancreatic tissue is identified in the wall of the lower esophagus. This finding represents an example of which of the following congenital tumor-like conditions? (A) Choristoma (B) Hamartoma (C) Hemangioma (D) Papilloma (E) Teratoma

(A) Choristoma Choristomas are minute or microscopic aggregates of normal tissue in aberrant locations. Choristomas are represented by rests of pancreatic tissue in the wall of the gastrointestinal tract or of adrenal tissue in the renal cortex. By contrast, hamartomas (choice B) represent focal, benign overgrowths of one or more of the mature cellular elements of a normal tissue, often with one element predominating. Hemangiomas (choice C) are the most frequently encountered tumors in childhood. Dx: Choristoma

A 12-month-old boy is brought to the emergency room for examination of his right arm following a tumble at home. Radiologic examination of the limb reveals a recent fracture of the right ulna and evidence of additional healing fractures. The child is noted to have blue sclerae. This patient most likely carries a mutation in a gene that encodes which of the following proteins? (A) Collagen (B) Fibrillin (C) Keratin (D) Myosin (E) Tubulin

(A) Collagen Osteogenesis imperfecta (OI), or brittle bone disease, is a group of inherited disorders expressed principally as fragility of bone. The genetic defects in the four types of OI are heterogeneous, but all affect the synthesis of type I collagen. Type I OI is characterized by a normal appearance at birth, but fractures of many bones occur during infancy and at the time the child learns to walk. Such patients have been described as being as "fragile as a China doll." Children with type I OI typically have blue sclerae as a result of the deficiency in collagen fibers, which imparts translucence to the sclera. A high incidence of hearing loss occurs because fractures and fusion of bones of the middle ear restrict their mobility. Fibrillin gene mutations (choice B) are found in patients with Marfan syndrome. Keratin gene mutations (choice C) are found in patients with epidermolytic hyperkeratosis. Dx: Osteogenesis imperfecta

A neonate is born with severe motor dysfunction involving the lower extremities. Radiologic studies show that vertebral bodies in the lumbar region lack posterior arches. The vertebral defects are covered by a thin membrane. The space underneath the membrane contains a mass of tissue that is composed of meninges and spinal cord. The parents ask for information regarding risks for similar birth defects in their future offspring. You mention that dietary supplementation of the maternal diet has been shown to reduce the incidence of neural tube defects. What is this substance? (A) Folic acid (B) Niacin (C) Thiamine (D) Vitamin B6 (E) VitaminB12

(A) Folic acid Spina bifida is a congenital defect in the closure of the spinal canal. Like other dysraphic disorders (anencephaly, meningocele, and meningomyelocele), spina bifida is of polygenic origin. Folic acid supplied in the periconceptional period lowers the incidence of neural tube defects. In 1998, the United States Food and Drug Administration began requiring manufacturers of enriched flour, bread, and some other products to supplement these foods with folate. This mandate has been associated with a significant decrease in the incidence of neural tube defects. Folic acid deficiency can result in elevated serum levels of homocysteine, a maternal risk factor for neural tube defects. Thiamine deficiency (choice C) causes beri-beri (polyneuropathy, edema, and heart failure). Vitamin B12 deficiency (choice E) causes megaloblastic anemia but not neural tube defects. Dx: Spina bifida

The Cystic Fibrosis patient described in the previous question carries mutations in the gene that encodes which of the following types of protein? (A) Membrane ion channel (B) Mitochondrial transport protein (C) Na+/K+ ATPase (D) Nuclear transport protein (E) Receptor tyrosine kinase

(A) Membrane ion channel The gene responsible for cystic fibrosis (CF) encodes a large protein termed the cystic fibrosis transmembrane conductance regulator (CFTR). CFTR is a member of the ATP-binding family of membrane transporter proteins that constitutes a chloride channel in most epithelia. The secretion of chloride anions by mucous- secreting epithelial cells controls the parallel secretion of fluid and, consequently, the viscosity of the mucus. It is estimated that 1 in 25 whites is a heterozygous carrier of the CF gene, and the incidence is 1 in 2,500 newborns. The most common cause of morbidity and mortality in patients with CF is pulmonary disease, secondary to chronic infections. Receptor tyrosine kinase gene mutations often lead to uncontrolled cell growth. Dx: Cystic fibrosis

A 28-year-old woman gives birth to a stillborn with a severe neural tube defect (neonate shown in the image). This birth defect was caused by an error of morphogenesis that occurred at which of the following stages of development after fertilization? (A) 1 to 10 days (B) 20 to 40 days (C) 90 to 120 days (D) 6 to 9 months (E) Birth trauma

(B) 20 to 40 days Anencephaly refers to the congenital absence of the cranial vault, with cerebral hemispheres either missing or reduced to small masses. It is a dysraphic anomaly of neural tube closure that results from an injury to the fetus between the 23rd and 26th day of gestation. During fetal development, the neural plate is transformed into the neural tube by fusion of the posterior surfaces. Failure of the neural tube to close results in the lack of closure of the overlying bony structures of the cranium and an absence of the calvarium, skin, and subcutaneous tissues of this region. The exposed brain is incompletely formed or absent. Blastocyst formation and implantation occur on days 1 to 10 after fertilization (choice A). Dx: Acrania, neural tube defect

A 16-year-old girl complains that she has not started menstruating like other girls her age. The patient is short (4 ft, 11 in) and has a thick-webbed neck. Physical examination reveals widely spaced nipples and poor breast development. If this patient's genetic disease was caused by nondisjunction during mitosis of a somatic cell in the early stages of embryogenesis, which of the following is the patient's most likely karyotype? (A) 45,X (B) 45,X/46,XX (C) 45,X/46,XY (D) 47,XX,+21 (E) 47,XXY

(B) 45,X/46,XX Mitotic nondisjunction may involve embryonic cells during early stages of development and result in chromosomal aberrations. This condition in which the body contains two or more karyotypically different cell lines is called mosaicism. Mosaicism involving sex chromosomes is found in patients with Turner and Klinefelter syndromes. Turner syndrome refers to the spectrum of abnormalities that result from the presence of complete or partial monosomy of the X chromosome in a phenotypic female. Half of women with Turner syndrome lack an entire X chromosome (monosomy X, choice A). The remainder of women with Turner syndrome are mosaics or display structural aberrations of the X chromosome. Mosaics characterized by a 45,X/46,XX karyotype (15%; choice B) tend to have milder phenotypic manifestations. In about 5% of patients, the mosaic karyotype is 45,X/46,XY (choice C). A patient with the 47,XX,+21 karyotype (choice D) is a female with Down syndrome. The 47,XXY karyotype (choice E) is a feature of Klinefelter syndrome. Dx: Turner syndrome

A 25-year-old pregnant woman, at 16 weeks of gestation, visits her obstetrician. A screening test suggests the possibility of a neural tube defect in her fetus. An ultrasound examination shows a 3-cm neural tube defect in the thoracic spine. The screening test that was administered to the mother measured serum levels of which of the following proteins? (A) Albumin (B) Alpha-fetoprotein (C) Bilirubin (D) Chromogranin (E) Human chorionic gonadotropin

(B) Alpha-fetoprotein Screening of preg- nant women for serum AFP and examination by ultrasonog- raphy allow detection of virtually all anencephalic fetuses. Levels of the other proteins are not significantly affected by a neural tube defect in the fetus. Diagnosis: Neural tube defect, spina bifida

The mother of a newborn boy is alarmed that her baby regurgitates at every feeding. An endoscopic examination reveals that the child's esophagus is almost completely occluded. This finding represents an example of which of the following errors of morphogenesis? (A) Aplasia (B) Atresia (C) Dysplasia (D) Dysraphicanomaly (E) Ectopia

(B) Atresia Atresia refers to defects caused by the incomplete formation of a lumen. Many hollow organs originate as strands and cords of cells whose centers are programmed to die, thus forming a central cavity or lumen. Atresia of the esophagus is characterized by partial occlusion of the lumen, which was not fully established in embryogenesis. Dysplasia (choice C) is caused by abnormal organization of cells into tissues, which is a situation that results in abnormal histogenesis. Tuberous sclerosis is a striking example of dysplasia, in which the brain contains aggregates of normally developed cells arranged into grossly visible "tubers." Ectopia (choice E) is an anomaly in which an organ is outside its normal anatomic site. Dx: Esophageal atresia

A 4-year-old boy is found to have extremely pliable skin. His parents note that he bruises easily. His joints can be hyperextended. Biochemical studies demonstrate a deficiency of lysyl hydroxylase. Ultrastructural examination of a skin biopsy of this patient would most likely reveal abnormalities associated with which of the following cell/ tissue components? (A) Actin-myosin filaments (B) Collagen fibers (C) Glycocalyx (D) Intermediate filaments (E) Mitochondria

(B) Collagen fibers Ehlers-Danlos syndromes (EDS) are a group of rare, autosomal dominant, inherited disorders of connective tissue that feature remarkable hyperelasticity and fragility of the skin, joint hypermobility, and often a bleeding diathesis. The common feature of most types of EDS is a generalized defect in collagen, including abnormalities in its molecular structure, synthesis, secretion, and degradation. Patients typically can stretch the skin many centimeters, and trivial injuries can lead to serious wounds. Because sutures do not hold well, dehiscence of surgical incisions is common. Hypermobility of the joints allows unusual extension and flexion. Abnormalities would not be expected in the other cell/tissue components listed. Dx: Ehlers-Danlos syndrome

The patient in the previous question delivers a female baby at 37 weeks of gestation with evidence of severe generalized edema (neonate shown in the image) due to Erythroblastosis fetalis. The baby is given exchange transfusions with Rh-negative cells but subsequently dies. Which of the following best describes the pathogenesis of anasarca in this baby? (A) Bilirubin encephalopathy (B) Congestive heart failure (C) Nephrotic syndrome (D) Respiratory distress syndrome (E) Rupture of the liver

(B) Congestive heart failure Erythroblastosis fetalis is a hemolytic disease of the newborn caused by maternal antibodies against fetal erythrocytes. Erythroblastosis fetalis does not ordinarily occur during the first pregnancy, because the quantity of fetal blood necessary to sensitize the mother is introduced into her circulation only at the time of delivery, too late to affect the fetus. However, when the sensitized mother again bears an Rh-positive fetus, much smaller quantities of fetal D antigen can elicit an increase in IgG antibody titer. This cycle is exaggerated in multiparous women, and the severity of erythroblastosis tends to increase progressively with each succeeding pregnancy. However, even after multiple pregnancies, only 5% of Rh-negative women are ever delivered of infants with erythroblastosis fetalis. The severity of erythroblastosis fetalis varies from a mild hemolysis to fatal anemia, and the pathological findings are determined by the extent of the hemolytic disease. Hydrops fetalis refers to the most serious form of erythroblastosis fetalis, and is characterized by severe edema secondary to congestive heart failure caused by severe anemia. The other choices do not cause of anasarca in erythroblastosis fetalis. Dx: Erythroblastosis fetalis, hydrops fetalis

A 27-year-old woman presents for a pregnancy test. She recalls drinking heavily during the week in which she may have conceived. What is the most likely consequence of toxic exposure to the conceptus during early (preimplantation) development? (A) Conjoined twins (B) Embryonic lethality (C) Placenta accreta (D) Neural tube defect (E) Ventricular-septal defect

(B) Embryonic lethality If a conceptus is exposed to harmful exogenous influences, the noxious agent exerts the same effect on all blastomeres and also causes death. Thus, either a conceptus dies or development proceeds uninterrupted, since the interchangeable blastomeres replace the loss. As a rule, exogenous toxins acting on preimplantation- stage embryos do not produce errors of morphogenesis and do not cause malformations. The most common consequence of toxic exposure at the preimplantation stage is death of the embryo, which often passes unnoticed or is perceived as heavy, albeit delayed, menstrual bleeding. Approximately 30% of fertilized ova are aborted spontaneously, without the woman being aware that pregnancy had occurred. Placenta accreta (choice C) is an abnormal adherence of the placenta to the underlying uterine wall. The other choices are errors of morphogenesis that manifest at later stages of development. Dx: Spontaneous abortion

A 25-year-old woman complains of recurrent bone pain and increasing abdominal girth. Physical examination reveals massive hepatosplenomegaly. Radiologic studies reveal several radiolucent bone defects. A bone marrow biopsy discloses enlarged cells with a fibrillar appearance reminiscent of "wrinkled tissue paper." Microscopic examination of a splenectomy specimen is shown. This patient most likely carries mutations in the gene that encodes which of the following types of hydrolytic enzymes? (A) Galactosidase (B) Glucosidase (C) Hexokinase (D) N-acetylgalactosaminidase (E) Neuraminidase

(B) Glucosidase Gaucher disease is characterized by the accumulation of glucosylceramide, primarily in the lysosomes of macrophages. The underlying abnormality in Gaucher disease is a deficiency in glucocerebrosidase, a type of lysosomal acid β-glucosidase. The hallmark of this disorder is the presence of Gaucher cells, which are lipid- laden macrophages that are characteristically present in the red pulp of the spleen, liver sinusoids, lymph nodes, lungs, and bone marrow. These cells are derived from the resident macrophages in the respective organs (e.g., Kupffer cells in the liver and alveolar macrophages in the lung). Galactosidase gene mutations (choice A) are found in patients with Fabry disease. N-acetylgalactosaminidase gene mutations (choice D) are found in patients with Tay-Sachs disease. Dx: Gaucher disease

A 4-year-old boy is brought to the physician by his parents because he tires easily. Physical examination reveals weakness in the pelvic and shoulder girdles and enlargement of the child's calf muscle. Serum levels of creatine kinase are elevated. A biopsy of calf muscle shows marked variation in size and shape of muscle fibers. There are foci of muscle fiber necrosis, with myophagocytosis, regenerating fibers, and fibrosis. Molecular diagnostic assays would most likely show alterations in the length of the primary transcript for which of the following muscle-associated proteins? (A) Actin (B) Desmin (C) Dystrophin (D) Glycogenphosphorylase (E) Myosin

(C) Dystrophin Duchenne muscular dystrophy (DMD) is a severe, X-linked condition characterized by progressive degeneration of muscles, particularly those of the pelvic and shoulder girdles. A milder form of the disease is known as Becker muscular dystrophy (BMD). Both DMD and BMD are caused by a deficiency of dystrophin, a member of the family of membrane cytoskeletal proteins, which includes α-actinin and spectrin. The protein is located on the cytoplasmic face of the plasma membrane of muscle cells and is linked to it by integral membrane glycoproteins (dystrophin-associated glycoprotein complex), which in turn, are bound to extracellular laminin. It has been proposed that the absence of dystrophin leads to a defective membrane that is damaged during contraction, an effect that predisposes to death of the myocyte. Serum levels of creatine kinase are increased. Glycogen phosphorylase gene mutations (choice D) are found in patients with McArdle disease. Dx: Duchenne muscular dystrophy

A 42-year-old woman in her third pregnancy delivers a female neonate at 30 weeks of gestation. The baby develops jaundice within 2 days. The unconjugated serum bilirubin is 15 mg/dL. Which of the following is the most serious complication of untreated hyperbilirubinemia in this neonate? (A) Acutepancreatitis (B) Bronchopulmonarydysplasia (C) Encephalopathy (D) Gallstones (E) Hemolyticanemia

(C) Encephalopathy Kernicterus, also termed bilirubin encephalopathy, is defined as a neurological condition associated with severe jaundice and characterized by bile staining of the brain, particularly of the basal ganglia, pontine nuclei, and dentate nuclei in the cerebellum. Kernicterus (Ger- man: kern, nucleus) is essentially confined to newborns with severe unconjugated hyperbilirubinemia, usually related to erythroblastosis fetalis. The bilirubin derived from the destruction of erythrocytes and the catabolism of the released heme is not easily conjugated by the immature liver, which is deficient in glucuronyl transferase. The development of kernicterus is directly related to the level of unconjugated bilirubin and is rare in term infants when serum bilirubin levels are below 20 mg/ dL. Premature infants are more vulnerable to hyperbilirubinemia and may develop kernicterus at levels as low as 12 mg/dL. Bilirubin is thought to injure the cells of the brain by interfering with mitochondrial function. Severe kernicterus leads initially to loss of the startle reflex and athetoid movements, which in 75% progresses to lethargy and death. Most surviving infants have severe choreoathetosis and mental retardation; a minor- ity have varying degrees of intellectual and motor retardation. Exchange transfusions may keep the maximum serum bilirubin at an acceptable level. However, phototherapy, which converts the toxic unconjugated bilirubin into isomers that are nontoxic and excreted in the urine, has greatly reduced the need for exchange transfusions. The other choices are not complications of untreated hyperbilirubinemia in newborns. Dx: Kernicterus, physiological jaundice

A 25-year-old man with a history of autism and mental retardation is seen by a genetic counselor. The man has coarse facial features, an increased head circumference, and macro-orchidism. His maternal uncle is similarly affected. After further evaluation, a diagnosis of fragile X syndrome is rendered. What is the most likely underlying cause of this patient's genetic disease? (A) Chromosomalnondisjunction (B) Chromosomeinversion (C) Expansion of trinucleotide repeat (D) Frame-shiftmutation (E) Nonreciprocal translocation

(C) Expansion of trinucleotide repeat Fragile X syndrome, the most common cause of inherited mental retardation, is caused by expansion of a CGG trinucleotide repeat in a noncoding region immediately adjacent to the FMR1 gene on the X chromosome. In a poorly understood manner, the expanded CGG repeat silences the FMR1 gene by methylation of its promoter. The abnormal repeat is associated with an inducible "fragile site" on the X chromosome, which appears in cytogenetic studies as a nonstaining gap or chromosomal break. The male newborn afflicted with the fragile X syndrome appears normal, but during childhood, characteristic features appear, including an increased head circumference, facial coarsening, joint hyperextensibility, enlarged testes, and abnormalities of the cardiac valves. Mental retardation is profound, with IQ scores varying from 20 to 60. A significant proportion of autistic male children carry a fragile X chromosome. The other choices do not cause fragile X syndrome. Dx: Fragile X syndrome

33 A 28-year-old man presents to the emergency room 1 hour after experiencing crushing substernal chest pain. Laboratory studies and ECG confirm the diagnosis of acute myocardial infarction. The patient dies 24 hours later of cardiac arrhythmia. This patient most likely had which of the following genetic diseases? (A) Adult-onset (type 2) diabetes (B) α1-Antitrypsin deficiency (C) Familial hypercholesterolemia (D) Marfan syndrome (E) Niemann-Pickdisease

(C) Familial hypercholesterolemia Familial hypercholesterolemia is an autosomal dominant disorder caused by mutations of the gene encoding the LDL receptor. It is one of the most common autosomal dominant disorders, affecting 1 in 500 adults in the United States. The gene defect affects the uptake of LDL in the liver, causing hyper- cholesterolemia. Clinically, the disease presents as severe atherosclerosis, which usually becomes symptomatic at an early age. Diabetes mellitus (choice A) also causes accelerated atherosclerosis but rarely at this age. Marfan syndrome (choice D) is associated with dissecting aortic aneurysm. Niemann-Pick disease (choice E) is a hereditary lysosomal storage disease. Dx: Familial hypercholesterolemia

A 35-year-old pregnant woman delivers a baby prematurely at 28 weeks of gestation. Shortly after birth, the neonate becomes short of breath, with intercostal retraction and nasal flaring during respiration. The neonate is placed on a ventilator, but dies of respiratory insufficiency and intraventricular hemorrhage. Microscopic examination of the lungs at autopsy is shown. The eosinophilic material lining the air spaces represents an accumulation of which of the following proteins? (A) Collagen (B) Dystrophin (C) Fibrin (D) Fibronectin (E) Laminin

(C) Fibrin The pathogenesis of respiratory distress syndrome (RDS) of the newborn is intimately linked to a deficiency of surfactant. Collapse of the alveoli (atelectasis) secondary to surfactant deficiency results in perfused but not ventilated alveoli, a situation that leads to hypoxia and acidosis. The leak of fibrin-rich fluid into the alveoli from the injured vascular bed contributes to the typical clinical and pathologic features of RDS. On gross examination, the lungs are dark red and airless. The alveolar ducts are lined by conspicuous, eosinophilic, fibrin-rich, amorphous structures, termed hyaline membranes. Although collagen (choice A), fibronectin (choice D), and laminin (choice E) are found in most tissues, they do not represent the major protein found in hyaline membranes. Dx: Respiratory distress syndrome of the neonate

A 34-year-old woman in her second pregnancy delivers a female neonate with severe generalized edema and jaundice. A CBC of the neonate shows hemolytic anemia. Subsequent workup of the mother and the newborn reveal an Rh-incompatibility. Transplacental passage of which of the following proteins is the principal cause of anasarca and jaundice in this neonate? (A) ComplementC3 (B) ComplementC4 (C) IgG (D) IgM (E) Interferon-α

(C) IgG Erythroblastosis fetalis is an antibody-mediated hemolytic disease that affects the fetus in utero. It is usually caused by transplacental passage of maternal antibodies to antigens expressed on fetal RBCs. The introduction of Rh-positive fetal erythrocytes (>1 mL) into the circulation of an Rh-negative mother at the time of delivery sensitizes her to the D antigen. When the antigen-sensitized mother again bears an Rh-positive fetus, much smaller quantities of fetal D antigen elicit an increase in antibody titer. In contrast to IgM (choice D), IgG antibodies are small enough to cross the placenta and thus produce hemolysis in the fetus. This cycle is exaggerated in multiparous women, and the severity of erythroblastosis tends to increase progressively with each succeeding pregnancy. Dx: Hemolytic anemia of the neonate, erythroblastosis fetalis

A 22-year-old man complains about his inability to conceive a child. On physical examination, the patient is noted to be tall (6 ft, 5 in) and exhibits gynecomastia and testicular atrophy. Laboratory studies demonstrate increased serum levels of follicle-stimulating hormone. Cytogenetic studies reveal a chromosomal abnormality. What is the most common cause of this patient's chromosomal abnormality? (A) Expansion of a trinucleotide repeat (B) Isochromosome formation (C) Meiotic nondisjunction (D) Nonreciprocal translocation (E) Ring chromosome formation

(C) Meiotic nondisjunction Klinefelter syndrome, or testicular dysgenesis, is related to the presence of one or more X chromosomes in excess of the normal male XY complement. Most persons with Klinefelter syndrome (80%) have one extra X chromosome (47,XXY karyotype). The additional X chromosome(s) arises as a result of nondisjunction during gametogenesis. In half of cases, nondisjunction occurs during paternal meiosis I, leading to a sperm containing both an X and a Y chromosome. Fertilization of a normal oocyte by such a sperm gives a zygote with a 47,XXY complement of chromosomes. Klinefelter syndrome occurs in 1 per 1,000 male newborns, which is roughly comparable to the incidence of Down syndrome. None of the other choices are associated with trisomy. Dx: Klinefelter syndrome

A 50-year-old man with a history of type 2 diabetes mellitus asks about the chances that his children will inherit this metabolic disorder. The patient is told that he has a genetic disease that shows which of the following patterns of inheritance? (A) Autosomaldominant (B) Autosomalrecessive (C) Multifactorial (D) X-linkeddominant (E) X-linkedrecessive

(C) Multifactorial Most of the common chronic disorders of adults represent multifactorial genetic diseases that tend to "run in families." Such maladies include diabetes, atherosclerosis, and many forms of cancer, arthritis, and hypertension. Fragile X syndrome and Duchenne-Becker muscular dystrophy are examples of X-linked recessive genetic diseases. Dx: Diabetes mellitus

The parents of a 2-year-old boy with hyposadias (urethra opens on the ventral aspect of the penis) visit a genetic counselor to discuss the chances that a similar birth defect will occur in their future offspring. This birth defect shows which of the following patterns of inheritance? (A) Autosomal recessive (B) Autosomal dominant (C) Multifactorial (D) X-linked dominant (E) X-linked recessive

(C) Multifactorial The inheritance of a number of birth defects is multifactorial. Most normal human traits are inherited neither as dominant nor as recessive mendelian attributes, but rather in a more complex manner. For example, multifactorial inheritance determines intelligence, height, skin color, body habitus, and even emotional disposition. Similarly, most of the common chronic disorders of adults represent multifactorial genetic diseases and are well known to "run in families." Such maladies include diabetes, atherosclerosis, and many forms of cancer and arthritis, and hypertension. The inheritance of a number of birth defects is also multifactorial (e.g., cleft lip and palate, pyloric stenosis, hypospadias, and congenital heart disease). The concept of multifactorial inheritance is based on the notion that multiple genes interact with various environmental factors to produce disease in an individual patient. Such inheritance leads to familial aggregation that does not obey simple mendelian rules (see choices A, B, D, and E). As a consequence, the inheritance of polygenic diseases is studied by the methods of population genetics, rather than by the analysis of individual family pedigrees. Dx: Hypospadias, multifactorial inheritance

If the neonate described in Question 24 had survived, which of the following would be the most likely complication related to anoxia and acidosis? (A) Bilirubin encephalopathy (kernicterus) (B) Erythroblastosis fetalis (C) Necrotizin genterocolitis (D) Pulmonary embolism (E) Ventricular septal defect

(C) Necrotizin genterocolitis The first symptom of RDS (usually appearing within an hour of birth) is increased respiratory effort, with forceful intercostal retraction and the use of accessory neck muscles. Despite advances in neonatal intensive care, the overall mortality of RDS is about 15%. Necrotizing enterocolitis is the most common acquired gastrointestinal emergency in newborns and is thought to be related to ischemia of the intestinal mucosa. This injury is followed by bacterial colonization, usually with Clostridium difficile. The lesions vary from those of typical pseudomembranous enterocolitis to gangrene and perforation of the bowel. None of the other choices are related to respiratory insufficiency. Dx: Respiratory distress syndrome of the neonate, enterocolitis

The Neurofibromatosis type 1patient (described in the previous question) is at increased risk of developing which of the following malignant neoplasms? (A) Ganglioneuroma (B) Glioblastomamultiforme (C) Neurofibrosarcoma (D) Serouscystadenocarcinoma (E) Squamous cell carcinoma

(C) Neurofibrosarcoma One of the major complications of neurofibromatosis type 1 (NF1), occurring in 3% to 5% of patients, is the appearance of a neurofibrosarcoma in a neurofibroma. NF1 is also associated with an increased incidence of other neurogenic tumors, including meningioma, optic glioma, and pheochromocytoma. The other tumors listed are not associated with NF1. Dx: Neurofibromatosis, type 1

A 10-month-old boy who was adopted from an orphanage in Eastern Europe presents for a physical examination. His parents believe that he is failing to meet developmental milestones. The child is fair skinned and has blond hair. On physical examination, the patient is noted to have a "mousy" odor. Laboratory studies demonstrate an inborn error of amino acid metabolism. To prevent mental retardation, this patient should be placed on a special diet that lacks which of the following essential amino acids? (A) Isoleucine (B) Methionine (C) Phenylalanine (D) Threonine (E) Tryptophan

(C) Phenylalanine Phenylketonuria (PKU, hyperphenylalaninemia) is an autosomal recessive disorder characterized by progressive mental deterioration in the first few years of life due to high levels of circulating phenylalanine, secondary to a deficiency of phenylalanine hydroxylase. The disorder is based on a genetic defect, but its expression depends on the provision of a dietary constituent. The affected infant appears normal at birth, but mental retardation is evident within a few months. Infants with PKU tend to have fair skin, blond hair, and blue eyes because the inability to convert phenylalanine to tyrosine leads to reduced melanin synthesis. These patients exude a "mousy" odor due to the formation of phenylacetic acid. The treatment of PKU involves the restriction of phenylalanine in the diet. None of the other essential amino acids accumulates in patients with PKU. Dx: Phenylketonuria

If the parents of the child described in the previous question have a total of four sons and two daughters, then, on average, how many of their children may be expected to be asymptomatic (i.e., silent) carriers of this gene mutation? (A) One child (B) Two children (C) Three children (D) Four children (E) Five children

(C) Three children Tay-Sachs disease is inherited as an autosomal recessive trait and is predominantly a disorder of Ashkenazi Jews, in whom the carrier rate is 1 in 30, and the natural incidence of homozygotes is 1 in 4,000 live newborns. In autosomal recessive diseases, on average, half of the offspring are expected to be heterozygotes and silent carriers of the gene mutation. Dx: Tay-Sachs disease

A 25-year-old man presents for a routine physical examination. The patient is tall (6 ft, 5 in) and has long fingers (shown in the image). One year later, he suffers a dissecting aortic aneurysm. This patient most likely carries a mutation in a gene that encodes which of the following proteins? (A) Collagen (B) Dystrophin (C) Elastin (D) Fibrillin (E) Myosin

(D) Fibrillin The cause of Marfan syndrome has been established as missense mutations in the gene coding for fibrillin-1 (FBN1). Fibrillin is a family of connective tissue proteins analogous to the collagens. It is widely distributed in many tissues in the form of a fiber system termed microfibrils. For example, the deposition of elastin on microfibrillar fibers produces the concentric rings of elastin in the aortic wall. Collagen gene mutations (choice A) are found in patients with Ehlers-Danlos syndrome and osteogenesis imperfecta. Dystrophin gene mutations (choice B) are found in patients with muscular dystrophy. Dx: Marfan syndrome

A 4-year-old boy is admitted to the hospital with pneumonia and respiratory distress. The nurses report that the child's bowel movements are greasy and have a pungent odor. A sweat- chloride test is positive. Which of the following mechanisms of disease is the most likely cause of steatorrhea in this child? (A) Abnormal dietary intake (B) Bacterial overgrowth (C) Hyperbilirubinemia with kernicterus (D) Lack of pancreatic enzyme secretion (E) Obstruction caused by meconium ileus

(D) Lack of pancreatic enzyme secretion Cystic fibrosis (CF) is an autosomal recessive disorder affecting children, which is characterized by (1) chronic pulmonary dis- ease, (2) deficient exocrine pancreatic function, and (3) other complications of inspissated mucus in a number of organs, including the small intestine, the liver, and the reproductive tract. The diagnosis of CF is most reliably made by the demonstration of increased concentrations of electrolytes in the sweat. The decreased chloride conductance characteristic of CF results in a failure of chloride reabsorption by the cells of the sweat gland ducts and, hence, to the accumulation of sodium chloride in the sweat. All of the pathologic consequences of CF can be attributed to the presence of abnormally thick mucus. Lack of pancreatic enzyme secretion in patients with CF causes malabsorption and foul-smelling fatty stools (steatorrhea). The other choices do not address the underlying cause of malabsorption in patients with CF. Dx: Cystic fibrosis

A 30-year-old pregnant woman visits her obstetrician for prenatal care and eventual delivery. The patient volunteers that two of her three children had "yellow jaundice" at birth. Her youngest girl had been severely jaundiced and had been given two blood transfusions. Prenatal laboratory tests indicate that the mother is blood type O, Rh negative, whereas her husband is blood type A, Rh positive. The obstetrician samples amniotic fluid at 36 weeks of gestation to ascertain whether the fetus is mature enough for preterm delivery. Quantitative analysis of which of the following was most likely used as an indicator of fetal lung maturity? (A) Absorbance at 450 nm (B) Alpha-fetoprotein (C) Creatinine (D) Lecithin (E) Total protein

(D) Lecithin Immaturity of the lungs poses one of the most common and immediate threats to the viability of the low birth weight infant because the lining cells of the fetal alveoli do not differentiate until late pregnancy. Alveoli are maintained in the expanded state, in part, by the presence of pulmonary surfactant. This material, which is produced by type II pneumocytes, is a complex mixture of several phospholipids, 75% phosphatidylcholine (lecithin) and 10% phosphatidylglycerol. The concentration of lecithin increases rapidly at the beginning of the third trimester and, thereafter, rises rapidly to reach a peak near term. Maturity of the fetal lung can be assessed by measuring pulmonary surfactant released into the amniotic fluid. A lecithin-to-sphingomyelin ratio above 2:1 implies that the fetus will survive without developing respiratory distress syndrome. Alpha-fetoprotein (choice B) is used to monitor for anencephaly. Dx: Erythroblastosis fetalis

An 18-year-old woman delivers a male neonate following a difficult labor and delivery (dystocia). The baby appears vigorous at birth (Apgar score = 9), but a cephalohematoma is apparent 2 hours later. A CT scan of the baby's head shows subperiosteal hemorrhage over one of the calvarial bones. What is the most likely outcome of this complication of labor and delivery? (A) Facial nerve palsy (B) Kernicterius (C) Respiratory distress syndrome (D) Spontaneous resolution (E) Subarachnoid hemorrhage

(D) Spontaneous resolution Birth injury spans the spectrum from mechanical trauma to anoxic damage. Some birth injuries relate to poor obstetric manipulation, whereas many are unavoidable sequelae of routine delivery. Birth injuries occur in about 5 per 1,000 live births. Factors that predispose to birth injury include cephalopelvic disproportion, dystocia (difficult labor), prematurity, and breech presentation. Cephalohematoma is defined as a subperiosteal hemorrhage that is confined to a single cranial bone and becomes apparent within the first few hours after birth. It may or may not be associated with a linear fracture of the underlying bone. Most cephalohematomas resolve without complication and require no treatment. Dx: Cephalohematoma

The parents of an infant with cleft lip and palate (infant shown in the image) visit a genetic counselor to discuss the chance that a similar birth defect will occur in their future offspring. In addition to teratogen exposure and multifactorial inheritance, which of the following is an important cause of this error of morphogenesis? (A) Downsyndrome (B) First pregnancy before 25 years of age (C) Maternal-fetal Rh incompatibility (D) Structural chromosomal abnormality (E) Turnersyndrome

(D) Structural chromosomal abnormality Cleft lip and cleft palate exemplify multifactorial inheritance in which multiple genes interact with various environmental factors to produce disease. On the 35th day of gestation, the frontal prominence fuses with the maxillary process to form the upper lip. Disturbances in gene expression at this time (hereditary or environmental) lead to interference with proper fusion and result in cleft lip, with or without cleft palate. In addition to multifactorial inheritance, this developmental anomaly may be part of a malformation syndrome caused by teratogens (e.g., rubella and anticonvulsants). It is also often encountered in children with chromosomal abnormalities (correct answer). The incidence of cleft lip, with or without cleft palate, is 1 in 1,000, and the incidence of cleft palate alone is 1 in 2,500. If one child is born with a cleft lip, the chances are 4% that the second child will exhibit the same defect. If the first two children are affected, the risk of cleft lip increases to 9% for the third child. The more severe the anatomical defect, the greater the probability of transmitting cleft lip will be. Whereas 75% of cases of cleft lip occur in boys, the sons of women with cleft lip have a four times higher risk of acquiring the defect than the sons of affected fathers. None of the other choices are associated with a significantly increased risk of cleft lip. Dx: Cleft lip, multifactorial inheritance

A 5-year-old boy presents with a maculopapular rash. On physical examination, the rash affects the palms and soles. Cracks and fissures are noted around the mouth and anus. There is funduscopic evidence of interstitial keratitis. Mild hepatosplenomegaly is present. The anterior tibial bones exhibit an outward curvature. What is the most likely etiology of these clinicopathologic findings? (A) AIDS (B) Cytomegalovirus (C) Herpes (D) Syphilis (E) Toxoplasmosis

(D) Syphilis The acronym TORCH refers to a complex of similar signs and symptoms produced by fetal or neonatal infection with a variety of microorganisms, including Toxoplasma (T), Rubella (R), Cytomegalovirus (C), and Herpes (H). The letter "O" represents "others" including congenital syphilis. The acronym was coined to alert pediatricians to the fact that infections in the fetus and newborn by TORCH agents are usually indistinguishable from each other and that testing for one of the four major TORCH agents should include testing for the other three and others as well. The organism that causes syphilis, Treponema pallidum, is transmitted to the fetus by a mother who has acquired syphilis during pregnancy. A maculopapular rash is a common early finding in congenital syphilis. The most common osseous lesion in congenital syphilis is periostitis and outward curving of the anterior tibia (saber shins). Flat raised plaques (condylomata lata) around the anus and female genitalia may develop early or after a few years. The diagnosis of congenital syphilis is suggested by clinical findings and a history of maternal infection. None of the other pathogens cause these clinicopathologic findings. Dx: TORCH syndrome, syphilis

A 10-year-old child presents with xanthomas on the extensor surfaces of his forearms. Laboratory studies demonstrate a total serum cholesterol of 820 mg/dL. The child's mother and maternal grandfather also have elevated serum cholesterol. This patient most likely has mutations in the gene that encodes which of the following proteins involved in lipid metabolism? (A) ApoE4 (B) Cholesterolhydroxylase (C) Chylomicron transport protein (D) High-density lipoprotein receptor (E) Low-density lipoprotein receptor

(E) Low-density lipoprotein receptor Familial hypercholesterolemia is an autosomal dominant disorder characterized by high levels of LDLs in the blood, accompanied by the deposition of cholesterol in arteries, tendons, and skin. It is one of the most common autosomal dominant disorders, and in its heterozygous form, it affects at least 1 in 500 adults in the United States. Only 1 in 1 million persons is homozygous for the disease. Familial hypercholesterolemia results from abnormalities in the gene that encodes the cell surface receptor that removes LDLs from the blood. Mutations in the other genes do not cause hypercholesterolemia. Dx: Familial hypercholesterolemia

A 42-year-old woman gives birth to a neonate with multiple congenital abnormalities. Physical findings included a flat facial profile, slanted eyes, epicanthal folds, Brushfield spots, short nose, short neck, dysplastic ears, clinodactyly, a large protruding tongue, and a pronounced heart murmur. What is the most common cause of this developmental birth disease? (A) Chromosomal deletion (B) Chromosomal translocation (C) Expansion of trinucleotide repeat (D) Frame shift point mutation (E) Nondisjunction

(E) Nondisjunction Nondisjunction during the first meiotic division of gametogenesis accounts for most (92% to 95%) patients with Down syndrome who have trisomy 21. The extra chromosome 21 is of maternal origin in about 95% of Down syndrome children. Translocation of an extra long arm of chromosome 21 to another acrocentric chromosome (choice B) causes about 5% of cases of Down syndrome. The other choices are unrelated to trisomy 21. Dx: Down syndrome, trisomy 21

A 22-year-old primigravida notes absent fetal movement for 2 days. The fetus is delivered stillborn at 19 weeks' gestation. The macerated fetus shows marked hydrops fetalis and a large posterior cystic hygroma of the neck. At autopsy, internal anomalies include aortic coarctation and a horseshoe kidney. Which of the following karyotypes is most likely to be present in cells obtained from this fetus? A 45,X B 47,XX,+18 C 47,XX,+21 D 47,XYY E 69,XXX

A 45,X The findings listed are characteristic of Turner syndrome (monosomy X), which accounts for many first-trimester fetal losses. The hygroma is quite suggestive of this disorder. Fetuses with this finding are rarely live-born. Trisomy 18 can be marked by multiple anomalies, but over- lapping fingers and a short neck are more typical features. Down syndrome (47,XX,+21) may be accompanied by a hygroma and hydrops, but ventricular septal defect is more frequent than coarctation, and horseshoe kidney is uncommon. The 47,XXY karyotype (Klinefelter syndrome) does not result in stillbirth, and these males have no major congenital defects. Triploidy with 69 chromosomes typically leads to fetal loss, but hydrops and hygroma are not features of this condition.

A 25-year-old woman with amenorrhea has never had menarche. On physical examination, she is 145 cm (4 ft 9 in) tall. She has a webbed neck, a broad chest, and widely spaced nipples. Strong pulses are palpable in the upper extremities, but there are only weak pulses in the lower extremities. On abdominal MR imaging, her ovaries are small, elongated, and tubular. Which of the following karyotypes is she most likely to have? A 45,X/46,XX B 46,X,X(fra) C 47,XXY D 47,XXX E 47,XX,+16

A) 45,X/46,XX The features described are those of classic Turner syndrome. Individuals who reach adulthood may have mosaic cell lines, with some 45,X cells and some 46,XX cells. A female carrier of the fragile X syndrome, X(fra), is less likely to manifest the disease than a male, but the number of triple repeat sequences (CGG) increases in her male offspring. The 47,XXY karyotype occurs in Klinefelter syndrome; affected individuals appear as phenotypic males. The "superfemale" karyotype (XXX) leads to mild mental retardation. Trisomy 16 is a cause of fetal loss early in pregnancy.

21 An 11-year-old mentally retarded boy is able to carry out activities of daily living, including feeding and dressing himself. On physical examination, he has brachycephaly and oblique palpebral fissures with prominent epicanthal folds. A transverse crease is seen on the palm of each hand. On auscultation of the chest, there is a grade III/VI systolic murmur. Which of the following diseases is he most likely to develop by age 20 years? A Acute leukemia B Acute myocardial infarction C Aortic dissection D Chronic renal failure E Hepatic cirrhosis

A) Acute leukemia Down syndrome (trisomy 21) is one of the trisomies that can result in a live-born infant. Although children with Down syndrome can function well, they often have many associated congenital anomalies. Among the more common is congenital heart disease, including ventricular septal defect. There is also a tenfold to twentyfold increased risk of acute leukemia. Virtually all individuals with Down syndrome who live to age 40 years have evidence of Alzheimer disease. Myocardial infarction at a young age suggests familial hypercholesterolemia. Aortic dissection is seen in individuals with Marfan syndrome. Chronic renal failure may be seen in genetic disorders that produce polycystic kidneys. Hepatic cirrhosis is a feature of galactosemia.

A 15-year-old girl has developed multiple nodules on her skin over the past 10 years. On physical examination, there are 20 scattered, 0.3-cm to 1-cm, firm nodules on the patient's trunk and extremities. There are 12 light brown macules averaging 2 to 5 cm in diameter on the skin of the trunk. Slit-lamp examination shows pigmented nodules in the iris. A sibling and a parent are similarly affected. Genetic analysis shows a loss-of-function mutation. Which of the following inheritance patterns is most likely to be present in this family? A Autosomal dominant B Autosomal recessive C Mitochondrial D Multifactorial E X-linked recessive

A) Autosomal dominant Neurofibromatosis type 1 (NF-1) is characterized by the development of multiple neurofibromas and pigmented skin lesions. Neurofibromas are most numerous in the dermis but also may occur in visceral organs. Patients with NF-1 also may develop a type of sarcomatous neoplasm known as a malignant peripheral nerve sheath tumor (MPNST). NF-1 is a tumor suppressor that appears with an autosomal dominant pattern of inheritance, though some cases result from spontaneous new mutations (no prior family members with the mutation). NF-1 exhibits variable expressivity, because the manifestations (location and types of neoplasms) are not the same in all patients. The other forms of inheritance listed are not associated with tumor suppressor genes.

A 2-year-old child has had failure to thrive since birth, with progressive neurologic deterioration. On physical examination, the child has hepatosplenomegaly and lymphadenopathy. Laboratory studies show pancytopenia. A bone marrow biopsy is obtained and microscopically shows numerous pale phagocytic cells filled with fine vacuoles (secondary lysosomes). An abnormality in genetic encoding for which of the following types of protein is most likely causing this child's disease? A Enzyme B Growth regulation C Ion channel D Receptor E Structural support

A) Enzyme This child has Niemann-Pick disease type A, the worst form. Death typically ensues by age 3 years. This inborn error of metabolism occurs in a degradation pathway for sphingomyelin, and accumulation of the abnormal lipid intermediate leads to lysosomal storage disease, mainly affecting tissues of the mononuclear phagocyte system, as well as the central nervous system. Genes involved in growth regulation are typically protoncogenes and tumor suppressor genes, and mutations may underlie development of malignancies. Cystic fibrosis results from mutations in the CFTR gene that encodes chloride ion channels. Familial hypercholesterolemia is an example of a disease resulting from an abnormal LDL receptor. Many autosomal dominant conditions, such as Marfan syndrome from fibrillin-1 gene mutations, involve structural protein abnormalities.

An infant born to a family living in Belarus appeared normal at birth, but at 6 months is noted to have worsening motor incoordination and mental obtundation. On examination, the infant has retinal pallor with a prominent macula. Flaccid paralysis develops by 1 year of age. Based on the figure, which of the following mutations most likely occurred? A Frameshift B Nonsense C Point D Three-base pair deletion E Trinucleotide repeat

A) Frameshift The infant has Tay-Sachs disease, an autosomal recessive condition that has a gene frequency higher in some populations, including Ashkenazi Jews and Quebecois. The gene that encodes for the enzyme hexosaminidase A in the diagram has a four-base pair insertion, leading to an altered reading frame (frameshift) and appearance of a stop codon that prematurely terminates reading to produce a nonfunctional enzyme. A frameshift mutation changes the remaining sequence of amino acids in a protein. A point mutation may change the codon to the sequence of a stop codon that truncates the protein being synthesized, typically leading to degradation of the protein. A point mutation typically is a missense mutation that leads to replacement of just one amino acid for another in the protein chain; this can lead to abnormal conformation and function of the protein. A deletion of three base pairs leads to loss of a single amino acid in a protein. A three-base pair deletion, as occurs in cystic fibrosis, results in a frameshift involving just a single amino acid. A trinucleotide repeat is the inheritance pattern for fragile X syndrome, which is caused by triple repeat expansions in the FMR1 gene.

A 3-year-old boy has had progressive developmental delay, ataxia, seizures, and inappropriate laughter since infancy. The child has a normal karyotype of 46,XY, but DNA analysis shows that he has inherited both of his number 15 chromosomes from his father. These findings are most likely to be indicative of which of the following genetic mechanisms? A Genomic imprinting B Maternal inheritance pattern C Mutation of mitochondrial DNA D Trinucleotide repeat expansion E X-linked inheritance pattern

A) Genomic imprinting This child has features of Angelman syndrome, and the DNA analysis shows uniparental disomy. The Angelman gene encoded on chromosome 15 is subject to genomic imprinting. It is silenced on the paternal chromosome 15, but is active on the maternal chromosome 15. If the child lacks maternal chromosome 15, there is no active Angelman gene in the somatic cells. This gives rise to the abnormalities typical of this disorder. The same effect occurs when there is a deletion of the Angelman gene from the maternal chromosome 15. The other listed options do not occur in uniparental disomy.

A 23-year-old woman, G2, P1, gives birth at 37 weeks to a small-for-gestational-age male infant. The left hand of an infant is shown in the figure. During the pregnancy, fetal ultrasound showed an endocardial cushion defect and polyhydramnios from probable duodenal atresia. Which of the following chromosomal abnormalities is most likely to be present? A 45,X B 47,XX,+21 C 47,XY,+18 D 69,XXY E 47,XXY

B) 47,XX,+21 The figure shows a single palmar flexion crease and a single flexion crease on the fifth digit, both features of trisomy 21. Although there is an increased risk of Down syndrome with increasing maternal age, most infants with Down syndrome are born to younger women because there are far more pregnancies at younger maternal ages. Monosomy X may be marked by a short fourth metacarpal. With trisomy 18, the fingers are often clenched, with digits 2 and 5 overlapping digits 3 and 4. Triploidy may be marked by syndactyly of digits 3 and 4. There are no characteristic hand features in males with Klinefelter syndrome.

A 4-year-old girl has sudden onset of right hip pain. On examination, the child's right hip is dislocated. The child can bend her thumb backward to touch the forearm. Her skin is noted to be extraordinarily stretchable. Radiographs of her spine show marked lateral and anterior curvature. She develops retinal detachments later in childhood. A sibling is similarly affected. A mutation in tenascin-X is identified. Which of the following is the most likely cause for this child's findings? A Congenital syphilis B Deficient collagen synthesis C Diet lacking in vitamin D D Multiple congenital anomalies E Trauma from battering

B) Deficient collagen synthesis The joints are frequently involved in most variants of Ehlers-Danlos syndrome (EDS), and tensile strength is reduced so that skin is hyperextensible, and joints are hypermobile. Deficiency of the enzyme lysyl hydroxylase can lead to defects in types I and III collagen and is inherited as an autosomal recessive disorder. Kyphoscoliosis and ocular problems also are present in this type of EDS. When EDS-like features are present, but no collagen gene mutations identified, then abnormal tenascin-X, a large multimeric protein of extracellular matrix that affects synthesis and fibril formation of type VI and type I collagens may be present. Con- genital syphilis can produce abnormalities of bone, such as saber shin from periosteitis and perichondritis, but does not affect the skin. Vitamin D deficiency in childhood producing rickets is accompanied by bowing deformities of long bones, but not skin abnormalities. The pattern of findings here suggests a structural gene defect leading to development of abnormalities, and not congenital anomalies without a specific cause. Battered children typically have multiple contusions and fractures, but the skin and bone structure are normal.

A 9-month-old infant has had numerous viral and fungal infections since birth. On physical examination, no congenital anomalies are noted. Laboratory studies show hypocalcemia. FISH analysis of the infant's cells is performed. A metaphase spread is shown in the figure, with probes to two different regions on chromosome 22. Which of the following cytogenetic abnormalities is most likely to be present? A Hyperdiploidy B Deletion C Inversion D Monosomy E Translocation

B) Deletion The infant has DiGeorge syndrome, resulting from a chromosome 22q11.2 microdeletion. This is indicated in the metaphase spread by the presence of only three dots be- cause this region is deleted on one chromosome 22, but both number 22 chromosomes are present. Hyperdiploidy is more than 46 chromosomes. With aneuploidy, there is an abnormal number of chromosomes (trisomy, monosomy), and loss or gain of autosomes tends to produce fetal loss except for some cases of trisomies 13, 18, and 21 and monosomy X. A chromosome inversion would shift the marked region to a different part of the same chromosome. In monosomy, only one of a pair of chromosomes is present. A translocation is the swapping of genetic material between two chromosomes.

A 20-year-old man is examined by a new family physician who discovers numerous pigmented patches and pedunculated skin tumors on his chest. Biopsy of a tumor discloses a benign neoplasm derived from Schwann cells. Neither the patient's father nor mother shows signs of this disease. This patient most likely carries a mutation in a gene that encodes which of the following proteins? A Epidermal growth factor receptor B GTPase activating protein C NF-κB transcription factor D Protein kinase C E Ras protein p21

B) GTPase activating protein Neurofibromatosis type 1 (NF1) is characterized by (1) disfiguring neurofibromas, (2) areas of dark pigmentation of the skin (caféau lait spots), and (3) pigmented lesions of the iris (Lisch nodules). It is one of the more common autosomal dominant disorders. The NF1 gene has a high rate of mutation, and half of cases are sporadic rather than familial. The protein product, termed neurofibromin, is expressed in many tissues and belongs to a family of GTPase-activating proteins (GAPs), which inactivate ras protein (choice E). Thus, NF1 is a classic tumor suppressor gene. Loss of GAP activity (in cells acquiring a second hit mutation) permits uncontrolled ras p21 activation, an effect that predisposes to the formation of benign neurofibromas. None of the other choices (A, C, and D) are associated with the pathogenesis of neurofibromatosis. Dx: Neurofibromatosis, type 1

A 10-year-old child has had recurrent otitis media for the past 8 years. On physical examination, there is hepatosplenomegaly. No external anomalies are present. Laboratory findings include anemia and leukopenia. A bone marrow biopsy is performed, and high magnification of the sample shows the findings depicted in the figure. An inherited deficiency of which of the following enzymes is most likely to produce these findings? A Alpha-1, 4-glucosidase B Glucocerebrosidase C Glucose-6-phosphatase D Hexosaminidase A E Lysyl hydroxylase

B) Glucocerebrosidase Gaucher disease type 1, seen in this child, ac- counts for 99% of cases and does not involve the central nervous system (CNS). It is caused by a deficiency of glucocerebrosidase, and infusion with this enzyme reduces severity and progression. Type 2 involves the CNS and is lethal in infancy. Type 3 also involves the CNS, although not as severely as type 2. A deficiency of alpha-1,4-glucosidase is a feature of Pompe disease. Von Gierke disease results from deficiency of glucose-6-phosphatase. Sphingomyelinase deficiency leads to Niemann-Pick disease types A and B. Type A, the more common form, is associated with severe neurologic deterioration. Type B, the less common form, may resemble the findings in this case, but the appearance of macrophages is different: they contain many small vacuoles. Tay-Sachs disease involves a deficiency of hexosaminidase A and is associated with severe mental retardation and death before 10 years of age. Lysyl hydroxylase deficiency is found in one form of Ehlers-Danlos syndrome.

Three female children in a family with the pedigree shown in the figure are noted to have histories of multiple fractures along with dental problems and hearing impairment. On examination, they are of normal height and weight for age, but have steel gray to blue sclerae. Both parents are unaffected by these abnormalities. Which of the following genetic ab- normalities is most likely to account for the findings in these children? A Genomic imprinting B Gonadal mosaicism C Multifactorial inheritance D Random X inactivation E Spontaneous new mutation

B) Gonadal mosaicism The appearance of multiple siblings with a similar condition known to be autosomal dominant, such as osteogenesis imperfecta type I, when both parents are phenotypically normal, suggests that one parent has a mutation confined to gonadal germ cells. In this case the affected parent is a mosaic, and somatic cells do not carry the mutation so there is no phenotypic expression. Genomic imprinting in uniparental disomy is a feature of Angelman syndrome. Multifactorial inheritance is a feature of diseases without a defined inheritance pattern, not osteogenesis imperfecta. Random X inactivation may account for females exhibiting features of an X-linked recessive condition, such as hemophilia A, but this is unlikely to occur three times in the same generation. A spontaneous new mutation can account for one child having an autosomal dominant condition that a parent does not, but the rarity of this event makes three such events in a generation highly unlikely.

A 1-year-old female infant has failure to thrive, poor neurologic development, and poor motor function. Physical examination shows a "cherry red" spot on the macula of the retina. The infant's muscle tone is poor. Both parents and a brother and sister are healthy, with no apparent ab- normalities. One brother with a similar condition died at the age of 18 months. This genetic disorder most likely resulted from a mutation involving a gene encoding for which of the following? A Mitochondrial enzyme B Lysosomal enzyme C Cell surface receptor protein D Structural protein

B) Lysosomal enzyme The findings listed suggest a severe inherited neurologic disease, and the pattern of inheritance (e.g., normal parents, an affected sibling) is consistent with an autosomal recessive disorder. This inheritance pattern and the cherry red spot in the retina are characteristic of Tay-Sachs disease, caused by mutations in the gene that encodes a lysosomal enzyme hexosaminidase A. Mitochondrial genes have a maternal pattern of transmission. Mutations in genes affecting receptor proteins and structural proteins typically give rise to an autosomal dominant pattern of inheritance.

The parents of a male infant report that male children over three generations in the mother's family have been affected by a progressive disorder involving multiple organ systems. These children have had coarse facial features, corneal clouding, joint stiffness, hepatosplenomegaly, and mental retardation, and many died in childhood. At autopsy, some of the children had subendothelial coronary arterial deposits that caused myocardial infarction. Laboratory testing of the infant now shows increased urinary excretion of mucopolysaccharides. Bone marrow biopsy is performed, and the accumulated mucopolysaccharides are found in macro- phages ("balloon cells" filled with minute vacuoles). Which of the following enzyme deficiencies is most likely to be seen in this infant? A Adenosine deaminase B α-l-Iduronidase C Glucocerebrosidase D Glucose-6-phosphatase E Hexosaminidase F Lysosomal glucosidase G Sphingomyelinase

B) α-l-Iduronidase Hunter syndrome, one of the mucopolysaccharidoses (MPS), results from deficiency of the lysosomal enzyme α-l- iduronidase. The glycosaminoglycans that accumulate in MPS include dermatan sulfate, heparan sulfate, keratan sulfate, and chondroitin sulfate. All of the MPS variants are autosomal recessive except for Hunter syndrome, which is X-linked recessive. Adenosine deaminase deficiency is a cause of severe combined immunodeficiency (SCID), an immunodeficiency state in which multiple recurrent infections occur after birth. Glucocerebrosidase deficiency is seen in Gaucher disease; in the most common form of the disease, there is no neurologic impairment, and patients have splenomegaly and skeletal disease as a consequence of increased lysosomal glucocerebrosides in cells of the mononuclear phagocyte system. Glucose-6-phosphatase deficiency leads to von Gierke disease, characterized by hepatomegaly, renomegaly, and impaired gluconeogenesis leading to hypoglycemia and hyperlipidemia. Hexosaminidase A deficiency occurs in Tay-Sachs disease; affected individuals manifest severe neurologic impairment, poor motor development, and blindness beginning in infancy. Lysosomal glucosidase deficiency, seen in Pompe disease, is associated with marked cardiomegaly and heart failure beginning in infancy. Sphingomyelinase deficiency occurs in Niemann-Pick disease type A, characterized by hepatosplenomegaly, lymphadenopathy, and severe motor and mental impairment.

A 27-year-old man and his 24-year-old wife have been trying to conceive a child for 6 years. Physical examination shows he has bilateral gynecomastia, reduced testicular size, reduced body hair, and increased length between the soles of his feet and the pubic bone. A semen analysis indicates oligospermia. Laboratory studies show increased follicle-stimulating hormone level and slightly decreased testosterone level. Which of the following karyotypes is this man most likely to have? A 46,X,i(Xq) B 47,XYY C 47,XXY D 46,XX/47,XX,+21 E 46,XY,del(22q11)

C 47,XXY Klinefelter syndrome is a relatively common chromosomal abnormality that occurs in about 1 of 660 live-born males. The findings can be subtle. The 46,X,i(Xq) karyotype is a variant of Turner syndrome (seen only in females), caused by a defective second X chromosome. The 47,XYY karyotype occurs in about 1 in 1000 live-born males and is associated with taller-than-average stature. A person with a mosaic such as 46,XX/47,XX,+21 has milder features of Down syndrome than a person with the more typical 47,XX,+21 karyotype. The 22q11 deletion syndrome is associated with con- genital defects affecting the palate, face, and heart and, in some cases, with T cell immunodeficiency.

A 39-year-old woman gives birth to a term infant with an umbilical hernia, Brushfield spots on the iris, macroglossia, low-set ears, oblique palpebral fissures, and a heart murmur. The infant survives to childhood and exhibits only mild mental retardation. Which of the following chromosomal abnormalities, affecting autosomes, is most likely to be present in the somatic cells of this child? A Haploidy B Monosomy C Mosaicism D Tetraploidy E Triploidy

C Mosaicism Though these features are characteristic of trisomy 21, the child is not severely affected, which suggests mosaicism. In mosaic individuals, greater numbers of potentially normal cells having the proper chromosomal complement are present, which may allow infants with abnormalities of chromosome number to survive to term and beyond. Haploidy is present in gametes. Loss of an autosomal chromosome is devastating; the only monosomy associated with possible survival to term is Turner syndrome (monosomy X). Most aneuploid conditions (trisomies and monosomies) lead to fetal demise; fetuses with trisomy 21 are the most likely to survive to term. Triploid fetuses rarely survive beyond the second trimester and are virtually never live-born. Likewise, tetraploidy accounts for many first-trimester fetal losses and is not survivable.

A 38-year-old woman gives birth at 35 weeks' gestation to a female infant. Physical examination of the infant soon after delivery shows rocker-bottom feet, a small face and mouth, and low-set ears. On auscultation of the chest, a heart murmur is detected. The appearance of the infant's hands is shown in the figure. The infant dies at 4 months of age. Which of the following karyotypes was most likely present in this infant? A 45,X B 46,XX C 47,XX,+18 D 47,XX,+21 E 48,XXX

C) 47,XX,+18 Trisomy 18 (Edwards syndrome), in which survival is shortened significantly, has this spectrum of findings. Turner syndrome (45,X) is associated with the presence of cystic hygroma and hydrops fetalis. The severe anomalies described in this case make it unlikely that a normal 46,XX karyotype is present. Down syndrome (47,XX,+21) is associated with longer survival than described in this case, and the external features can be quite subtle at birth. The "super- female" karyotype (XXX) leads to mild mental retardation. Generally, abnormal numbers of sex chromosomes are tolerated better than abnormalities of autosomes.

A 12-year-old boy has a cough and earache for the past 2 days. He has a history of recurrent infections, including otitis media, diarrhea, and pneumonia. Physical examination shows an erythematous right tympanic membrane, a cleft palate, and murmur suggestive of congenital heart disease. A thoracic CT scan shows a small thymus. Results of laboratory studies suggest mild hypoparathyroidism. Which of the following diagnostic studies is most likely to be helpful in diagnosing this patient's condition? A Adenosine deaminase assay in lymphocytes B Branched DNA assay for HIV-1 RNA level C FISH analysis with a probe for chromosome 22q11.2 D Lymph node biopsy E PCR analysis for trinucleotide repeats on the X chromosome

C) FISH analysis with a probe for chromosome 22q11.2 DiGeorge syndrome is an immunodeficiency characterized by infection, a small thymus, congenital malformations, and hypoparathyroidism. This cluster is characteristic of the 22q11.2 deletion syndrome, readily diagnosed by FISH. Adenosine deaminase deficiency can cause immunodeficiency, but it is not associated with congenital malformations. Branched DNA assay can detect HIV infection that can lead to AIDS, but no congenital anomalies are associated with this condition. A lymph node biopsy may show a reduction in T cells or B cells associated with various forms of immunodeficiency, but this is not a specific test that can aid in confirming a specific diagnosis. Trinucleotide repeats of the X chromosome, detected by PCR, are seen in fragile X syndrome, which manifests with mental retardation in males.

A 23-year-old woman gives birth to a term infant after an uncomplicated pregnancy. On physical examination, the infant has ambiguous external genitalia. The parents want to know the infant's sex, but the physician is hesitant to assign a sex without further information. A chromosomal analysis indicates a karyotype of 46,XX. An abdominal CT scan shows bilaterally enlarged adrenal glands, and the internal genitalia appear to consist of uterus, fallopian tubes, and ovaries. This clinical picture is most consistent with which of the following conditions? A Androgen insensitivity syndrome B Excessive trinucleotide repeats C Female pseudohermaphroditism D Mitochondrial DNA mutation E Nondisjunctional event with loss of Y chromosome

C) Female pseudohermaphroditism Physicians must be cautious in assigning sex to an infant with ambiguous genitalia; changing one's opinion is about as popular as an umpire changing the call. True hermaphroditism, with ovarian and testicular tissue present, is very rare. This infant has female pseudohermaphroditism, resulting from exposure of the fetus to excessive androgenic stimulation, which in this case is due to congenital adrenal hyperplasia. The gonadal and the karyotypic sex are female. Male pseudohermaphroditism has various forms, but the most common is androgen insensitivity; affected individuals are phenotypically females, but have testes and a 46,XY karyotype. Nondisjunctional events lead to monosomies or trisomies, and most result in fetal loss. Trinucleotide repeats are seen in males with fragile X syndrome. Abnormalities of mitochondrial DNA have a maternal transmission pattern and do not involve sex chromosomes or sexual characteristics.

A 22-year-old man has a sudden loss of vision in the right eye. On physical examination, there is a subluxation of the right crystalline lens. On auscultation of the chest, a mid- systolic click is audible. An echocardiogram shows a floppy mitral valve and dilated aortic arch. The patient's brother and his cousin are similarly affected. He is prescribed a beta-blocker. A genetic defect involving which of the following substances is most likely to be present in this patient? A Collagen B Dystrophin C Fibrillin-1 D NF1 protein E Spectrin

C) Fibrillin-1 Marfan syndrome is an autosomal dominant condition that is most often caused by qualitative defects in fibrillin from missense mutations in the fibrillin (FBN1) gene. An abnormal collagen gene can cause osteogenesis imperfecta and Ehlers-Danlos syndrome. Genetic mutations in the dystrophin gene are involved in Duchenne and Becker muscular dystrophies. The NF1 protein is abnormal in neurofibromatosis type 1. Disordered spectrin causes hereditary spherocytosis.

In a study of inheritance of the cystic fibrosis gene (CFTR), the genetic mutations in carriers and affected individuals are documented. Based on these findings, investigators determine that there is no simple screening test to detect all carriers of mutations of the CFTR gene. Which of the following is most likely to be the greatest limitation to development of a screening test for CFTR mutations? A Both copies of the gene must be abnormal for detection B Fluorescence in situ hybridization is labor-intensive and expensive C Frequency of mutations among ethnic groups limits sensitivity D Less than 1 individual in 10,000 is a heterozygote E Most mutations cannot be detected by PCR

C) Frequency of mutations among ethnic groups limits sensitivity When a genetic disease (e.g., cystic fibrosis) is caused by many different mutations, with different frequencies among populations, there is no simple screening test that can detect all the mutations. Although 70% of patients with cystic fibrosis have a 3-base pair deletion that can be readily detected by PCR (the ∆F508 mutation), the remaining 30% have disease caused by several hundred allelic forms of CFTR. To detect all would require sequencing of the CFTR genes. This prohibits mass screening. The other listed options do not apply.

A 32-year-old woman has had three pregnancies, all ending in stillbirths in the first trimester. On physical examination, she and her only spouse for all pregnancies have no abnormalities. Which of the following laboratory tests is most appropriate to perform on this woman for elucidating potential causes for recurrent fetal loss? A Genome-wide association study B Fluorescence in situ hybridization C Karyotyping D PCR analysis E Tandem mass spectroscopy

C) Karyotyping Recurrent fetal loss suggests a parental cause. A chromosomal abnormality such as a Robertsonian translocation may account for stillbirths, particularly in the first trimester, when many fetal losses result from chromosomal abnormalities. A genome-wide associations study applies to populations, not individuals, and establishes linkages of common diseases such as hypertension and diabetes mellitus to polygenic risks. Fluorescence in situ hybridization (FISH) analysis is useful for establishing chromosome number and morphology, such as translocations, inversions, and deletions, but is not as definitive as a karyotype. PCR analysis is useful for identifying specific gene defects, not whole chromosomal abnormalities. Tandem mass spectroscopy is used in newborn screening for biochemical inborn errors of metabolism, such as phenylketonuria.

A clinical study is undertaken with subjects from families in which complications of atherosclerotic cardiovascular disease and tendinous xanthomas occurred before age 30 years. Some of the children in these families are observed to have early atheroma formation. These affected individuals benefit from treatment with pharmacologic agents that inhibit HMG-CoA reductase. Affected individuals in these families are most likely to have a mutation in a gene encoding a cell surface receptor for which of the following? A Cortisol B Insulin C LDL cholesterol D Leptin E TGF-α

C) LDL cholesterol Familial hypercholesterolemia results from mutations in the LDL receptor gene, causing plasma LDL cholesterol to increase because it is not catabolized or taken up by the liver. It is an autosomal dominant disorder with a carrier rate of 1 in 500, so the frequency of homozygosity is 1 in 1 million. Heterozygotes have total serum cholesterol levels twice normal; homozygotes have levels even higher, with death from myocardial infarction by the second decade. The statin drugs inhibit the HMG-CoA reductase and reduce cholesterol levels in heterozygotes. Steroid hormone receptors, such as those for cortisol, are located in the cell nucleus. Insulin receptors play a role in glucose metabolism and glycemic control that may be part of diabetes mellitus with risk for atherosclerosis; statin drugs have no effect on diabetes mellitus. Abnormal leptin receptors may play a role in some forms of obesity. TGF-α is a growth factor with a role in inflammation, cell proliferation, and repair.

A 22-year-old woman delivers an apparently healthy female infant after an uncomplicated pregnancy. By 4 years of age, the girl has progressive, severe neurologic deterioration. Physical examination shows marked hepatosplenomegaly. A bone marrow biopsy specimen shows numerous foamy vacuolated macrophages. Analysis of which of the following factors is most likely to aid in the diagnosis of this condition? A Level of α1-antitrypsin in the serum B Level of glucose-6-phosphatase in hepatocytes C Level of sphingomyelinase in splenic macrophages D Number of LDL receptors on hepatocytes E Rate of synthesis of collagen in skin fibroblasts

C) Level of sphingomyelinase in splenic macrophages The clinical features of this child—neurologic involvement, hepatosplenomegaly , and accumulation of foamy macrophages—suggest a lysosomal storage disorder. One such disorder, with which the clinical history is quite compatible, is Niemann-Pick disease type A. It is characterized by lysosomal accumulation of sphingomyelin owing to a severe deficiency of sphingomyelinase. Globules of α1-antitrypsin are seen in the liver cells of individuals with inherited deficiency of α1-antitrypsin. The glycogen storage disease known as von Gierke disease results from glucose-6-phosphatase deficiency. In familial hypercholesterolemia, there are fewer LDL receptors on hepatocytes, leading to early and accelerated atherosclerosis by young adulthood. Collagen synthesis is impaired in individuals with Ehlers-Danlos syndrome.

A clinical study is performed involving complex genetic traits such as hypertension, heart disease, and diabetes. The study makes use of naturally occurring variations in DNA sequences that are found in exons and introns and are frequent and stable. Which of the following genetic markers is being used in this study? A Proto-oncogenes B Robertsonian translocations C Single-nucleotide polymorphisms D Three-base pair frameshift deletions E Trinucleotide repeat mutations

C) Single-nucleotide polymorphisms Single-nucleotide polymorphisms occur at a frequency of approximately 1 nucleotide in every 1000-base pair stretch and can be used in linkage analysis for identifying haplotypes associated with disease. Proto-oncogenes are genes encoding for proteins involved in cellular growth; mutant alleles of proto-oncogenes are called oncogenes and play a role in neoplasia. Robertsonian translocations involve portions of two chromosomes that trade places, but are not completely lost, and are balanced; carriers may not be affect- ed, but gametes have the potential to produce monosomies and trisomies. Frameshift deletions are a form of mutation that can lead to abnormal proteins and to conditions such as cystic fibrosis. Trinucleotide repeats found in DNA are three- nucleotide sequences that repeat multiple times and can be amplified to cause some disease conditions, such as fragile X syndrome.

34 A pregnant woman with a family history of fragile X syndrome undergoes prenatal testing of her fetus. PCR analysis to amplify the appropriate region of the FMR1 gene is attempted using DNA from amniotic fluid cells, but no amplified products are obtained. Which of the following is the most appropriate next step? A Routine karyotyping of the amniotic fluid cells B Routine karyotyping of the unaffected father C Southern blot analysis of DNA from the amniotic fluid cells D PCR analysis of the mother's FMR1 gene E No further testing

C) Southern blot analysis of DNA from the amniotic fluid cells Failure to find amplified product by PCR analysis in such a case could mean that the fetus is not affected, or that there is a full mutation that is too large to be detected by PCR. The next logical step is a Southern blot analysis of genomic DNA from fetal cells. Routine karyotyping of the amniotic fluid cells is much less sensitive than a Southern blot analysis. Karyotyping of the unaffected father cannot provide information about the status of the FMR1 gene in the fetus because amplification of the trinucleotide occurs during oogenesis. For the same reason, PCR analysis of the mother's FMR1 gene is of no value.

A female infant born at term shows failure to thrive and failure to achieve developmental milestones. A pedigree reveals only this child is affected out of four generations on both sides of the family. Tissue fibroblasts obtained from this child shows a 46,XX karyotype. Cultured fibroblasts show accumulation of an intermediate product in a metabolic pathway in which multiple enzymes are involved. What is the most likely recurrence risk for this condition in siblings of this infant? A 3% B 8% C 15% D 25% E 50% F 100%

D) 25% Most inborn errors of metabolism involve mutations in genes encoding for enzymes. Because one active allele produces half the needed enzyme, this is likely sufficient to avoid dis- ease. Inheritance of two mutant alleles, one from each parent, is required for appearance of disease, so the pattern is autosomal recessive, and the recurrence risk is 25%. Most autosomal recessive genes are infrequent in the population, so a family history is unlikely. Even if 1 in 10 persons carries the mutant recessive gene, a homozygote will be 1 in 400. The standard recurrence risk for any pregnancy is 3%. The recurrence risk is increased to 7% in diseases such as diabetes mellitus, or when a syndrome is identified without a defined inheritance pat- tern, or with multifactorial inheritance. Autosomal dominant conditions usually result from mutations in genes encoding for structural genes and have a recurrence risk of 50%.

A 36-year-old woman gives birth at 34 weeks' gestation to a male infant who lives for only 1 hour after delivery. On physical examination, the infant is at the 30th percentile for height and weight. Anomalies include microcephaly, a cleft lip and palate, scalp defects, and the finding shown in the figure. Which of the following karyotypes is most likely to be present in this infant? A 45,X B 46,XY C 47,XXY D 47,XY,+13 E 47,XY,+18 F 69,XXY

D) 47,XY,+13 Features of trisomy 13 (Patau syndrome) include cleft lip and palate, along with microcephaly, scalp defects, and postaxial polydactyly (an extra digit, shown in the figure). These infants also commonly have severe heart defects, and may also have cyclopia and holoprosencephaly. For mono-somy X (45,X) to be considered, the infant must be female. The severe anomalies described in this case may occur with a normal karyotype (46,XY), but the spectrum of findings, particularly the polydactyly, suggests trisomy 13. Klinefelter syndrome (47,XXY) results in phenotypic males who are hard to distinguish from males with a 46,XY karyotype. Infants with trisomy 18 lack polydactyly and are more likely to have micrognathia than are infants with trisomy 13. Triploidy with 69 chromosomes leads to stillbirth in virtually all cases.

A 27-year-old primigravida has a fetal ultrasound performed at 18 weeks' gestation. The male fetus is mildly growth retarded. Multiple congenital anomalies are present, including ventricular and atrial septal defects, horseshoe kidney, and omphalocele. Amniocentesis is performed, and the fetal cells obtained are examined using FISH analysis using a probe for one of the chromosomes. Based on the findings shown in the figure, which of the following karyotypic abnormalities is most likely to be present in this fetus? A 45,X/46,XX B 46,XY,del(22q11) C 46,XY,der(14;21)(q10.0),+21 D 47,XY,+18 E 47,XXY

D) 47,XY,+18 The infant has findings associated with trisomy 18. In the FISH analysis shown, the chromosomes in each cell have been painted with a marker for chromosome 18. In this case, there are three markers per cell, consistent with a trisomy. In reality, many cells would have to be counted to allow for artifacts in preparation. In most cases, trisomy 18 results from nondisjunctional events. Most infants with trisomy 18 are stillborn, and survival beyond 4 months is rare. The other listed options do not account for this FISH analysis or for this spectrum of anomalies.

A 19-year-old man has experienced failing eyesight and progressive muscle weakness for the past 7 years. Family history reveals that several of the patient's male and female relatives have similar symptoms. His mother, her brother and sister, and two of the aunt's children are affected, but the uncle's children are not. Which of the following types of genetic dis- orders is most likely to be present in this patient? A Trinucleotide repeat expansion B Genetic imprinting C X-linked inheritance pattern D Mitochondrial mutation E Uniparental disomy

D) Mitochondrial mutation This is a classic pattern of maternal inheritance resulting from a mutation in mitochondrial DNA. Males and females are affected, but affected males cannot transmit the disease to their offspring. Because mitochondrial DNA encodes many enzymes involved in oxidative phosphorylation, mutations in mitochondrial genes exert their most deleterious effects on organs most dependent on oxidative phosphorylation, including the central nervous system and muscles. The other listed options do not exhibit strict maternal inheritance.

A 13-year-old boy has been drinking large quantities of fluids and has an insatiable appetite. He is losing weight and has become more tired and listless for the past month. Laboratory findings include normal CBC and fasting serum glucose of 175 mg/dL. His parents, two brothers, and one sister are healthy. A maternal uncle is also affected. Which of the following is the probable inheritance pattern of his disease? A Autosomal dominant B Autosomal recessive C Mitochondrial DNA D Multifactorial E X-linked recessive

D) Multifactorial Type 1 diabetes mellitus has an increased frequency in some families, but the exact mechanism of inheritance is unknown. The risk is increased for offspring when first-order relatives are affected. HLA-linked genes and other genetic loci and environmental factors are considered important. This pattern of inheritance is multifactorial. The other listed inheritance patterns are not seen with most cases of diabetes mellitus.

An 8-year-old girl experiences sudden severe dyspnea. On examination, she has upper airway obstruction from soft tissue swelling in her neck. A radiograph shows a hematoma com- pressing the trachea. Laboratory studies show her prothrombin time (PT) is normal, but her partial thromboplastin time (PTT) is increased. Further testing reveals less than 1% of normal factor VIII activity. Both parents and two female siblings are unaffected by this problem, but a male sibling has experienced a similar episode. Which of the following genetic abnormalities is most likely to account for the findings in this girl? A Autosomal dominant mutation B Genomic imprinting C Germline mosaicism D Random X inactivation E Spontaneous new mutation

D) Random X inactivation This girl has features of hemophilia A. This X-linked recessive condition is expected to occur in males who inherit the one maternal X chromosome with the genetic mutation, and they do not have another X chromosome with a normal functional allele, as is the case in her brother. Hemophilia in a female can be explained by the Lyon hypothesis, which states that only one X chromosome in a female is active (the "turned off" X chromosome is the Barr body) for most genes, but this inactivation is a random event. Some unlucky fe- males are out on the tail end of the Poisson distribution of random events and have few active X chromosomes with the normal allele, leading to markedly diminished factor VIII ac- tivity. The other choices do not explain this phenomenon.

Multiple members of a family have a disease that is associated with a genetic change that involves substitution of adenine for thymine involving one base pair on homologous chromosomes. What is the best term to describe this finding? A Copy number variation B Deletion C Epigenetic change D Single nucleotide polymorphism E Trinucleotide repeat mutation F RNA alteration

D) Single nucleotide polymorphisms (SNPs) are found in less than 0.5% of the genome, and only 1% of these are found in coding regions that affect protein synthesis. Some of these account for point mutations that may be associated with disease conditions. C number variations (CNVs) involve variations in large contiguous regions of DNA from 1000 to a million base pairs. Epigenetic changes involve modulation of gene expression without any change in the DNA. Trinucleotide repeats involve increased numbers of base pairs. RNA alterations may modulate DNA expression, such as noncoding micro RNAs.

A case-control study is performed involving persons diagnosed with essential hypertension. Genetic analysis reveals linkage disequilibrium. Haplotypes of affected persons differ from the controls in the chromosome containing the angiotensinogen gene. Which of the following types of genetic analysis is most likely to yield this information? A Fluorescence in situ hybridization B Giemsa banded karyotyping C RNA expression D Single nucleotide polymorphisms E Southern blotting

D) Single nucleotide polymorphisms The human genome can be divided into blocks of DNA with varying numbers of contiguous single nucleotide polymorphisms (SNPs) that form haplotypes and can cluster from linkage disequilibrium, so that similar haplotypes suggest shared inheritance. The use of chips with more than a million SNPs can identify small variations in DNA from person to person, and linkage of these polymorphisms to disease can help narrow the search for candidate genes whose altered function may relate to a disease. Fluorescence in situ hybridization (FISH) probes aid in identifying specific chromosome regions and can identify abnormalities such as chromosomal deletions and translocations. The standard karyotype with banding provides information about chromosome number and major structural alterations, but does not identify specific genes or their loci. Analysis of mRNA expression provides a roundabout way of determining derivative DNA alterations, but DNA is easier to work with directly. Southern blotting has been largely supplanted by other techniques, but is useful for detection of trinucleotide repeat expansions and clonal gene rearrangements.

A pedigree reveals that multiple family members over four generations have been affected by the onset of congestive heart failure within the first four decades of life. A cardiomyopathy is suspected, but specific features of the disease are not known, and no prior genetic testing has been performed. Which of the following techniques involving DNA sequencing is most likely to identify a specific mutation in a cost-effective manner? A Pyrosequencing B Sanger sequencing C Targeted sequencing D Whole exome sequencing E Whole genome sequencing

D) Whole exome sequencing Whole exome sequencing limits the search for the roughly 2% of the genome that consists of protein-encoding exons responsible for as many as 80% of mendelian diseases, and the cost is significantly reduced compared to whole genome sequencing. Pyrosequencing is a more sensitive variation of Sanger sequencing that is most often used when testing for particular sequence variants in a background of normal alleles, such as tumor cells admixed with large numbers of stromal cells. Sanger sequencing is used together with PCR to allow sequencing of any known segment of DNA. Targeted sequencing is useful for identification of a single gene or panel of genes, either by subselecting relevant clones from a whole genome library via custom complementary probes, or by alternate preparations from genomic DNA such as multiplex PCR.

20 A healthy 20-year-old woman, G3, P2, Ab1, has previously given birth to a liveborn infant and a stillborn infant, both with the same karyotypic abnormality. On physical examination, she is at the 50th percentile for height and weight. She has no physical abnormalities noted. Which of the following karyotypic abnormalities is most likely to be present in this woman? A Deletion of q arm - del(22q) B Isochromosome - 46,X,i C Paracentric inversion - inv(18) D Ring chromosome - r(13) E Robertsonian translocation - t(14;21)

E )Robertsonian translocation t(14;21) Almost all of the normal genetic material is present in the case of a Robertsonian translocation because only a small amount of the p arm from each translocated chromosome is lost. The maternal karyotype is 45,XX,t(14;21). Statistically, one of six fetuses in a mother who carries a Robertsonian translocation will also be a carrier. In balanced reciprocal translocation, the same possibility of inheriting the defect exists. The other listed structural abnormalities are likely to result in loss of significant genetic material, reducing survivability, or to interfere with meiosis.

A 6-month-old male infant has failure to thrive and abdominal enlargement. His parents are concerned that he has shown minimal movement since birth. On physical examination, the infant has marked muscle weakness and hepatosplenomegaly . A chest radiograph shows marked cardiomegaly. He dies of congestive heart failure at age 19 months. The microscopic appearance of myocardial fibers at autopsy is shown in the figure. A deficiency of which of the following enzymes is most likely to be present in this infant? A Glucocerebrosidase B Glucose-6-phosphatase C Hexosaminidase A D Homogentisic acid oxidase E Lysosomal glucosidase F Sphingomyelinase

E) Lysosomal glucosidase Pompe disease is a form of glycogen storage dis- ease that results from a deficiency in lysosomal glucosidase (alpha-1,4-glucosidase). The glycogen stored in the myocardium results in massive cardiomegaly and heart failure within 2 years. Glucocerebrosidase deficiency occurs in Gaucher disease. In the most common form of the disease, there is no neurologic impairment, and patients have splenomegaly and skeletal disease as a con- sequence of increased lysosomal glucocerebrosides in cells of the mononuclear phagocyte system. Glucose- 6-phosphatase deficiency leads to von Gierke disease, characterized by hepatomegaly, renomegaly, and impaired gluconeogenesis leading to hypoglycemia and hyperlipidemia. Hexosaminidase A deficiency occurs in Tay-Sachs disease and is associated with severe neurologic impairment, poor motor development, and blindness beginning in infancy. Homogentisic acid oxidase deficiency leads to alkaptonuria with ochronosis or to deposition of a blue-black pigment in joints, resulting in arthropathy. Sphingomyelinase deficiency occurs in Niemann-Pick disease; affected individuals with type A have hepatosplenomegaly, lymphadenopathy, and severe motor and mental impairment.

A 25-year-old woman stops going to her aerobic exercise class because of severe muscle cramps that have occurred during every session for the past 2 months. Four hours after each session, she notices that her urine is a brown color. On physical examination, she has normal muscle development and strength. An inherited defect in which of the following substances is most likely to explain her findings? A Dystrophin B Fibrillin C Glucose-6-phosphatase D Lysosomal glucosidase E Muscle phosphorylase F Spectrin

E) Muscle phosphorylase McArdle disease is a form of glycogen storage disease in which a deficiency of muscle phosphorylase enzyme causes glycogen to accumulate in skeletal muscle. Onset is in young adulthood. Because strenuous exercise requires glycogenolysis and use of anaerobic metabolism, muscle cramps ensue, but the blood lactate level does not rise. Myoglobinuria is seen in about half of cases. A lack of dystrophin, a protein that stabilizes muscle membrane, characterizes Duchenne muscular dystrophy. A fibrillin gene mutation can lead to Marfan syndrome. Glucose-6-phosphatase deficiency leads to von Gierke disease, characterized by hepatomegaly, renomegaly, and impaired gluconeogenesis leading to hypoglycemia and hyperlipidemia. Lysosomal glucosidase deficiency is seen in Pompe disease, characterized by marked cardiomegaly and heart failure beginning in infancy. Abnormal spectrin, a RBC membrane cytoskeletal protein, leads to a condition known as hereditary spherocytosis.

A 66-year-old man has been prescribed clopidogrel to help prevent future acute coronary events. He states that his father, aunt, and brother took this drug and had adverse side effects, including excessive bleeding, when prescribed the recommended dose. Which of the following genetic tests will help determine the most appropriate dose of clopidogrel for this man? A Enumeration of tandem repeat sequences B Expression profiling of mRNA C FISH analysis for the karyotype D Measurement of an enzyme level E Probing for a cyp450 polymorphism

E) Probing for a cyp450 polymorphism Pharmacogenomic testing may reveal polymorphisms that affect drug metabolism. The cytochrome P-450 system in hepatocytes is involved in metabolism of many substances, including drugs. Detection of polymorphisms can provide information about modification of drug dosing, or the need for a different drug. Trinucleotide repeat mutations may affect gene expression, such as the huntingtin gene, but unlikely those involved in drug metabolism. Changes in DNA lead to alterations in mRNA expression, but detecting the DNA alterations directly tends to be a more effective testing strategy. FISH analysis is useful in identifying altered regions of chromosomes, not single genes abnormalities. Autosomal recessive conditions may affect enzyme levels, such G6PD, but polymorphisms may not affect enzyme or substrate levels that can be reliably detected.

7 An 11-year-old child has exhibited poor wound healing, even with minor trauma, since infancy. On examination she has hyperextensible joints and fragile, extremely stretchable skin. A diaphragmatic hernia was repaired soon after birth. One parent and one of three siblings are also affected. A mutation in a gene encoding for which of the following type of proteins is most likely causing this child's disease? A Enzyme B Growth regulation C Ion channel D Receptor E Structural support

E) Structural support This classical form of Ehlers-Danlos syndrome (EDS) results from an abnormality in collagen synthesis, and lack of normal collagen affects connective tissues in skin, bone, eye, and vasculature. There are multiple forms of EDS from different mutations affecting different aspects of collagen synthesis. The inheritance pattern described here is autosomal dominant, typical for inherited defects in structural proteins. Inborn errors of metabolism involving enzymes typically have an autosomal recessive pattern of inheritance, because half of gene function is sufficient to prevent disease. Genes involved in growth regulation are typically protooncogenes and tumor suppressor genes, and mutations may underlie development of malignancies. Cystic fibrosis results from mutations in the CFTR gene that encodes chloride ion channels. Familial hypercholesterolemia is an example of a disease resulting from an abnormal LDL receptor.

A study of families with fragile X syndrome reveals that 20% of affected men are carriers, but do not develop mental retardation. Genomic sequencing shows that these men have premutations having 55 to 200 trinucleotide repeats in the FMR1 gene. Half of these men exhibit a progressive neurodegenerative disease with cerebellar signs after age 50. Through which of the following mechanisms is their disease mediated? A Loss of mRNA transcription B Methylation with gene silencing C Random inactivation of the X chromosome D Reduced translation of the FMR1 protein E Toxicity from gain of function

E) Toxicity from gain of function FMR1 protein (FRMP) normally acts as a translation regulator to reduce protein synthesis at synaptic junctions. Thus a reduction in FMRP (loss of function) in the classic fragile X syndrome results in increased protein translation from mRNAs to alter synaptic activity and cause mental retardation. In the fragile X tremor/ataxia syndrome, the FMR1 gene is not methylated and silenced but continues to be transcribed, and CGG-containing FMR1 mRNAs accumulate in the neuronal nucleus and form intranuclear inclusions of aggregated mRNA that sequester RNA-binding proteins, leading to events that are toxic to the cell. Inactivation of the X chromosome occurs when more than one X chromosome is present, such as in a normal female or Klinefelter male.

Mental retardation has affected several generations of a family, and most of the affected individuals have been males. The severity of mental retardation has increased with each passing generation. Genetic testing is performed, and about 20% of the males who have the genetic abnormality are unaffected. Which of the following mechanisms is most likely to produce this genetic condition? A Frameshift mutation B Missense mutation C Mitochondrial DNA mutation D Point mutation E Trinucleotide repeat mutation

E) Trinucleotide repeat mutation Fragile X syndrome is a condition in which there are 250 to 4000 tandem repeats of the trinucleotide sequence CGG. Generally, as the number of trinucleotide repeats increases, the manifestations of the associated conditions worsen or have an earlier onset. The trinucleotide mutations are dynamic; because their number increases during oogenesis, subsequent male offspring have more severe disease compared with earlier generations. With a frameshift mutation, one, two, or three nucleotide base pairs are inserted or deleted. As a result, the protein transcribed is abnormal. A missense mutation results from a single nucleotide base substitution, and it leads to elaboration of an abnormal protein. Abnormalities of mitochondrial DNA, typically involving genes associated with oxidative phosphorylation, are transmitted on the maternal side. A point mutation of a single base pair may affect a single protein.

A 2-year-old child with failure to thrive since infancy now exhibits a seizure. Physical examination shows hepatomegaly and ecchymoses of the skin. Laboratory studies show a blood glucose level of 31 mg/dL. A liver biopsy specimen shows cells filled with clear vacuoles that stain positive for glycogen. Which of the following conditions is most likely to produce these findings? A Hurler syndrome B McArdle disease C Pompe disease D Tay-Sachs disease E Von Gierke disease

E) Von Gierke disease With von Gierke disease, from deficiency of glucose-6-phosphatase, stored glycogen is not metabolized readily to glucose. Affected individuals have severe hypoglycemia, which leads to convulsions. Intracytoplasmic accumulations of glycogen occur mainly in the liver and kidney. In Hurler syndrome, the enzyme α-l-iduronidase is deficient. Affected children have skeletal deformities and a buildup of mucopolysaccharides in endocardium and coronary arteries, leading to heart failure. Another form of glycogen storage disease, McArdle disease, results from a deficiency of muscle phosphorylase and leads to muscle cramping. Cardiomegaly and heart failure mark Pompe disease, the type II form of glycogen storage dis- ease. Tay-Sachs disease is characterized by a deficiency in hexosaminidase A and results in severe neurologic deterioration.


Set pelajaran terkait

Strategy week 5 (strategic innovation, industry context)

View Set

CHAPTER 35 (ASSESSMENT OF MUSCULOSKELETAL FUNCTION)

View Set

J: Chapter 22- Neurodevelopmental Disorders

View Set

Stevenson, "The Nature of Ethical Disagreement"

View Set